USAMO
1985


Problem - 3514
$0\le a_1\le a_2\le a_3\le \cdots$ is an unbounded sequence of integers. Let $b_n = m$ if $a_m$ is the first member of the sequence to equal or exceed $n$. Given that $a_{19}=85$, what is the maximum possible value of $a_1+a_2+\cdots a_{19}+b_1+b_2+\cdots b_{85}$

report an error